Tải bản đầy đủ (.pdf) (60 trang)

Một số tính chất của hàm lồi và ứng dụng trong bất đẳng thức và cực trị

Bạn đang xem bản rút gọn của tài liệu. Xem và tải ngay bản đầy đủ của tài liệu tại đây (392.74 KB, 60 trang )

ĐẠI HỌC THÁI NGUYÊN
TRƯỜNG ĐẠI HỌC KHOA HỌC
ĐỖ THỊ VÂN ANH
MỘT SỐ TÍNH CHẤT CỦA HÀM
LỒI VÀ ỨNG DỤNG TRONG
BẤT ĐẲNG THỨC VÀ CỰC TRỊ
LUẬN VĂN THẠC SĨ TOÁN HỌC
THÁI NGUYÊN - 2015
ĐẠI HỌC THÁI NGUYÊN
TRƯỜNG ĐẠI HỌC KHOA HỌC
ĐỖ THỊ VÂN ANH
MỘT SỐ TÍNH CHẤT CỦA HÀM
LỒI VÀ ỨNG DỤNG TRONG
BẤT ĐẲNG THỨC VÀ CỰC TRỊ
LUẬN VĂN THẠC SĨ TOÁN HỌC
Chuyên ngành: PHƯƠNG PHÁP TOÁN SƠ CẤP
Mã số: 60 46 01 12
Giáo viên hướng dẫn
TS. NGUYỄN VĂN NGỌC
THÁI NGUYÊN, 2015
Mục lục
Lời cảm ơn iv
Mở đầu 1
1 Hàm lồi và các tính chất 3
1.1 Định nghĩa . . . . . . . . . . . . . . . . . . . . . . . . . . . . . . . . 3
1.2 Một số tính chất cơ bản của hàm lồi, lõm . . . . . . . . . . . . . . 4
1.3 Tính lồi, tính liên tục và tính khả vi của các hàm số . . . . . . . 6
2 Một số bất đẳng thức và cực trị của các hàm lồi 11
2.1 Bất đẳng thức Jensen . . . . . . . . . . . . . . . . . . . . . . . . . 11
2.2 Sử dụng hàm lồi chứng minh các bất đẳng thức cơ bản đối với
dãy số . . . . . . . . . . . . . . . . . . . . . . . . . . . . . . . . . . . 12


2.2.1 Bất đẳng thức AM-GM với trọng và bất đẳng thức AM-GM 13
2.2.2 Bất đẳng thức trung bình lũy thừa . . . . . . . . . . . . . . 13
2.2.3 Bất đẳng thức H
˝
older . . . . . . . . . . . . . . . . . . . . . 15
2.2.4 Bất đẳng thức tam giác Minkowski . . . . . . . . . . . . . . 16
2.2.5 Bất đẳng thức Young . . . . . . . . . . . . . . . . . . . . . . 17
2.3 Một số bài toán ứng dụng bất đẳng thức Jensen . . . . . . . . . . 17
2.3.1 Đại số và lượng giác . . . . . . . . . . . . . . . . . . . . . . 17
2.3.2 Hình học . . . . . . . . . . . . . . . . . . . . . . . . . . . . . 32
2.4 Một số bài toán ứng dụng bất đẳng thức Karamata . . . . . . . . 35
2.4.1 Bất đẳng thức Karamata . . . . . . . . . . . . . . . . . . . 35
2.4.2 Các bài toán áp dụng . . . . . . . . . . . . . . . . . . . . . 36
2.5 Bất đẳng thức Shapiro . . . . . . . . . . . . . . . . . . . . . . . . . 42
2.5.1 Dẫn luận . . . . . . . . . . . . . . . . . . . . . . . . . . . . . 42
2.5.2 Trường hợp n = 3 . . . . . . . . . . . . . . . . . . . . . . . . 43
2.5.3 Trường hợp n = 5 . . . . . . . . . . . . . . . . . . . . . . . . 44
ii
2.5.4 Trường hợp n = 6 . . . . . . . . . . . . . . . . . . . . . . . . 45
2.6 Cực trị của một lớp hàm lồi nhiều biến và ứng dụng . . . . . . . . 47
2.6.1 Cực trị của một lớp hàm lồi nhiều biến . . . . . . . . . . . 47
2.6.2 Bất đẳng thức Kantorovich và ứng dụng . . . . . . . . . . 48
Kết luận 52
Tài liệu tham khảo 53
iii
LỜI CẢM ƠN
Lời đầu tiên của khóa luận này, tôi xin bày tỏ lòng biết ơn sâu sắc nhất
tới người thầy kính mến TS. Nguyễn Văn Ngọc, đã tận tình hướng dẫn,
giúp đỡ tôi trong suốt quá trình làm và hoàn thiện luận văn.
Tôi xin chân thành cảm ơn các thầy, cô giáo khoa Toán, Trường Đại

học Khoa học - Đại học Thái Nguyên, phòng Đào tạo Trường Đại học
Khoa học, những người đã trực tiếp giảng dạy và giúp đỡ tôi trong quá
trình học tập tại trường cùng toàn thể bạn bè và người thân đã đóng góp
ý kiến, giúp đỡ, động viên tôi trong quá trình học tập, nghiên cứu và hoàn
thành luận văn này.
Tuy nhiên, do sự hiểu biết của bản thân và trong khuôn khổ của luận
văn thạc sỹ nên bản luận văn mới chỉ trình bày được một phần nào đó.
Do thời gian có hạn và năng lực có phần hạn chế nên chắc chắn luận văn
không tránh khỏi những thiếu sót. Kính mong nhận được ý kiến đóng góp
của các thầy cô và bạn bè đồng nghiệp để bản luận văn được hoàn chỉnh
hơn.
Xin chân thành cảm ơn.
Thái Nguyên, tháng 04 năm 2015
Học viên
Đỗ Thị Vân Anh
iv
Mở đầu
Lý thuyết của các hàm lồi có vị trị quan trọng trong Toán học vì nó liên
quan đến nhiều lĩnh vực của Toán học, như giải tích lồi, giải tích hàm, tối
ưu, quy hoạch, v.v và là đối tượng nghiên cứu của nhiều nhà toán học.
Chúng tôi cũng được biết hiện nay đã có nhiều luận án và luận văn bằng
tiếng Việt về hàm lồi, nhưng các luận văn này hầu hết là thuộc chuyên
ngành Toán ứng dụng, hay Toán giải tích.
Trong chương trình Toán học ở bậc phổ thông, bất đẳng thức Jensen
và các mở rộng của nó đối với các hàm lồi, hàm lõm là công cụ hữu hiệu
để chứng minh các bất đẳng thức, hay tìm cực trị của các hàm số, đặc biệt
là đối với các hàm số có tính đối xứng theo các biến. Tuy nhiên, các kiến
thức về hàm lồi lại chưa được dạy ở bậc phổ thông (ở thập niên 90 của thế
kỷ trước đã có giới thiệu về hàm lồi và hàm lõm trong Giải tích 12, nhưng
sau đó đã bỏ đi). Hiện nay, các kiến thức về hàm lồi vẫn được dạy cho các

học sinh giỏi tham gia các đội tuyển quốc gia hay quốc tế. Do đó, việc bồi
dưỡng và nâng cao kiến thức về hàm lồi và ứng dụng của người dạy Toán
ở bậc THPT là cần thiết và bổ ích. Đó là lý do tôi chọn đề tài này làm
luận văn khoa học trong chuyên ngành Phương pháp Toán sơ cấp.
Mục đích của luận văn gồm có:
1. Tìm hiểu và học tập về hàm số lồi, đặc biệt là bất đẳng thức Jensen,
bất đẳng thức Karamata và các mở rộng.
2. Trình bày chứng minh của các bất đẳng thức cơ bản như bất đẳng
thức của dãy số, như bất đẳng AM-GM, bất đẳng thức Cauchy- Schwartz,
bất đẳng thức Holder, v.v bằng phương pháp sử dụng các tính chất của
hàm lồi. Các chứng minh này còn ít được giới thiệu trong các sách chuyên
1
khảo về toán sơ cấp bằng tiếng Việt.
3. Sưu tầm và trình bày cách chứng minh một số bài toán nâng cao của
đại số và hình học về bất đẳng thức và cực trị bằng phương pháp sử dụng
bất đẳng thức Jensen và các tính chất khác của các hàm lồi.
Bố cục của luận văn gồm có: Mở đầu, hai chương nội dung, Kết luận
và Tài liệu tham khảo.
Chương 1: Hàm lồi và các tính chất, trình bày cơ sở lý thuyết của
hàm lồi ( hàm lõm), như định nghĩa hàm lồi (hàm lõm) và các tính chất
cơ bản của hàm lồi.
Chương 2: Một số bất đẳng thức và cực trị của các hàm lồi là
nội dung chính của luận văn. Chương này trình bày những ứng dụng của
bất đẳng thức Jensen và bất đẳng thức Karamata chứng minh các bất
đẳng thức từ cơ bản đến nâng cao chủ yếu về đại số và lượng giác, trong
đó có bất đẳng thức Shapiro. Các bài toán về bất đẳng thức hình học chỉ
chiếm vị trí khiêm tốn trong luận văn.
Ngoài các bài toán về bất đẳng thức, chương này còn xét vấn đề cực trị
của một lớp hàm lồi nhiều biến và một số bài toán liên quan.
2

Chương 1
Hàm lồi và các tính chất
Chương này trình bày cơ sở lý thuyết của hàm lồi ( hàm lõm), như
định nghĩa hàm lồi (hàm lõm) và các tính chất cơ bản của hàm lồi. Nội
dung của chương này được hình thành chủ yếu từ các tài liệu [1] và [6].
1.1 Định nghĩa
Nếu không có gì cụ thể, trong chương này chúng ta sẽ dùng ký hiệu
I(a, b) để ngầm định một trong bốn tập hợp (a, b), (a, b], [a, b), [a, b].
Định nghĩa 1.1. Cho I = I(a, b) ⊂ R và hàm số f : I −→ R được gọi là
hàm lồi trên I nếu ∀x, y ∈ I và ∃λ ∈ [0, 1] thì
f(λx + (1 −λy)) ≤ λf(x) + (1 −λ)f(y). (1.1)
Nếu bất đẳng thức trên là ngặt với x = y và λ ∈ (0, 1) thì ta nói hàm f
là hàm lồi chặt.
Hàm số f(x) được gọi là lõm trong khoảng nói trên, nếu bất đẳng thức
(1.1) có chiều ngược lại, tức là
f(λx + (1 −λy)) ≥ λf(x) + (1 −λ)f(y). (1.2)
Dưới đây là một số hàm lồi, lõm đơn giản.
Ví dụ 1.1. Xét một số hàm thường gặp sau đây
1) f(x) = c, g(x) = x là những hàm vừa lồi vừa lõm trên R.
2) f(x) = x
r
với r ≥ 1 là lồi trên (0, ∞), với 0 ≤ r < 1 là hàm lõm trên
3
(0, ∞).
3) f(x) = e
x
là hàm lồi chặt trên R.
4) f(x) =
1
x

r
là hàm lồi trên khoảng ), x < ∞, nếu r > 0. Khi r là số
nguyên âm lẻ r = −2m − 1 thì f(x) là hàm lõm trên −∞ < x < 0, còn
với r là số nguyên âm chẵn r = −2m, thì f(x) là hàm lồi trong khoảng
nói trên.
5) f(x) = logx (ln x > 0, x > 0) là hàm lõm trên khoảng (0, ∞).
Sử dụng tính chất để kiểm tra các hàm số. Ta thấy
f(x) = e
x
, x ∈ R,
f(x) = tanx, x ∈ (0,
π
2
),
f(x) = x
a
, a > 1, x ∈ (0, +∞)
là những hàm lồi chặt. Trong khi đó các hàm số sau
f(x) = log
a
x, a > 1, x ∈ (0, +∞),
f(x) = sinx, x ∈ (0, π),
f(x) = cox, (−
π
2
,
π
2
)
là những hàm lõm chặt.

1.2 Một số tính chất cơ bản của hàm lồi, lõm
Các tính chất sau đây của các hàm lồi, lõm được suy ra trực tiếp từ
định nghĩa.
Tính chất 1.1. Nếu f(x) là hàm lồi (lõm) trên I(a, b) thì −f(x) là hàm
lõm (lồi) trên I(a, b).
Tính chất 1.2. Nếu f(x) là hàm lồi trên I(a, b) và c = const, thì cf(x)
là hàm lồi nếu c > 0, là hàm lõm nếu c < 0.
Tính chất 1.3. Tổng hữu hạn các hàm lồi (lõm) trên I(a, b) là một hàm
lồi (lõm) trên I(a, b).
4
Tính chất 1.4. Cho f là lồi trên khoảng I(a, b). Chứng minh rằng với
x, y, z ∈ I, x < y < z có bất đẳng thức
f(y) − f(x)
y −x

f(z) − f(x)
z − x

f(z) − f(y)
z − y
. (1.3)
Chứng minh. Giả sử x, y, z ∈ I và x < y < z. Tồn tại λ ∈ (0, 1) sao cho
y = λx + (1 −λ)z. Khi đó
λ =
y −z
x −z

1 −λ =
x −y
x −z

.
Vì f là hàm lồi nên ta có
f(y) ≤
y −z
x −z
f(x) +
x −y
x −z
f(z).
Do đó
f(y) − f(x) ≤
y −x
x −z
f(x) +
x −y
x −z
f(z),
Hay
f(y) − f(x)
y −x

f(z) − f(x)
z − x
.
Tính chất 1.5. Nếu f(x) là hàm liên tục và lồi trên I(a, b) và nếu hàm
g(x) là lồi và đồng biến trên tập giá trị của f(x) thì g(f(x)) là hàm lồi
trên I(a, b).
Tính chất 1.6. Nếu f(x) là hàm liên tục và lõm trên I(a, b) và nếu g(x)
là hàm lồi và nghịch biến trên tập giá trị của f(x), thì g(f(x)) là hàm lõm
trên I(a, b).

Tính chất 1.7. Nếu f(x) là hàm liên tục và đơn điệu (đồng biến hay
nghịch biến) trên I(a, b) và nếu g(x) là hàm ngược của f(x) thì ta có các
kết quả sau đây
1. f(x) lõm, đồng biến khi và chỉ khi g(x) lồi đồng biến.
2. f(x) lõm, nghịch biến khi và chỉ khi g(x) lõm nghịch biến.
3. f(x) lồi, nghịch biến khi và chỉ khi g(x) lồi, nghịch biến.
5
Tính chất 1.8. ( Định lý Proviciu). Với mọi hàm lồi trên I(a, b) đều có
bất đẳng thức
f(x) + f(y) + f(z) + 3f

x + y + z
3

≥ 2f

x + y
2

+ 2f

y + z
2

+ 2f

z + x
2

. (1.4)

Tính chất 1.9. ( Định lý A. Lupas). Với mọi hàm lồi f(x) trên I(a, b),
với bộ số dương p, q, r và với mọi x, y, z ∈ I(a, b) có bất đẳng thức
pf(x) + qf(y) + rf(z) + (p + q + r)f

px + qy + rz
p + q + r


(p + q)f

px + qy
p + q

+ (q + r)f

qy + rz
q + r

+ (r + p)f

rz + px
r + p

. (1.5)
Tính chất 1.10. ( Định lý Vasile Cirtoaje). Với mọi hàm lồi f(x) trên
I(a, b) và a
1
, a
2
, , a

n
∈ I(a, b), n ≥ 3, ta luôn có bất đẳng thức sau
f(a
1
) + f(a
2
) + + f(a
n
)
+ n(n −2)f

a
1
= a
2
+ + a
n
n

≥ (n − 1)[f(b
1
) + f(b
2
) + + f(b
n
)] (1.6)
trong đó b
i
=
1

n −1

j=i
a
j
.
1.3 Tính lồi, tính liên tục và tính khả vi của các
hàm số
Mệnh đề 1.1. Mọi hàm lồi trên khoảng I = (a, b) là hàm liên tục tại mỗi
điểm trong của khoảng đó.
Chứng minh. Cố định x
0
∈ I và chọn δ
0
> 0 sao cho (x
0
−δ
0
, x
0
+ δ
0
) ⊂ I
Với ∀x ∈ (x
0
− δ
0
, x
0
+ δ

0
), định nghĩa hàm số g(x) =
f(x) −f(x
0
)
x −x
0
.
Vì f là hàm lồi nên g là hàm số tăng trên (x
0
−δ
0
, x
0
) và (x
0
, x
0
+ δ
0
). Do
đó f

(x
0
−0) tồn tại. Quan trọng ta thấy rằng f

(x
0
−0) hữu hạn. Từ đó

suy ra với x
0
−δ
0
< x < x
0
< y < x
0
+ δ
0
ta có g(x) ≤ g(y). Thật vậy, ta
6
có thể viết x
0
= λx + (1 −λ)y với λ =
y −x
0
y −x
.
Vì f là hàm lồi nên ta có
λ
f(x
0
) −f(x
x
0
− x
≤ (1 − λ)
f(y) − f(x
0

)
y −x
0
.
Theo cách biểu diễn của λ ta có g(x) ≤ g(y). Cố định 0 < ε < 1 khi đó
tồn tại δ < δ
0
sao cho x ∈ (x
0
− δ, x
0
) để



f(x) −f(x
0
)
x −x
0
− f

(x
0
− 0)



< ε.
Từ đó tồn tại µ > 0 sao cho x ∈ (x

0
− µ, x
0
), ta có |f(x) −f(x
0
)| < 2ε.
Do đó f liên tục trái tại x
0
. Tương tự f liên tục phải tại x
0
. Vậy f liên
tục tại x
0
.
Nhận xét 1.1. Theo kết quả trên nếu f : [a, b] −→ R là hàm lồi thì f bị
chặn. Ta chỉ ra rằng kết quả này sẽ không đúng nếu miền xác định của f
không compact.
Thật vậy, xét hàm số f : (0, 1] −→ R được định nghĩa f(x) = x
−1

hàm lồi nhưng không bị chặn trên (0, 1].
Định nghĩa 1.2. Cho hàm f : I −→ R. Ta nói f Lipschitz trên I nếu có
một hằng số dương M (hằng số Lipschitz) sao cho ∀x, y ∈ I,
|f(x) −f(y)| ≤ M|x − y|.
Hàm f gọi là Lipschitz địa phương nếu mọi tập compact K ⊂ I tồn tại
hằng số M
K
phụ thuộc K sao cho ∀x, y ∈ K,
|f(x) −f(y)| ≤ M
K

|x −y|.
Mệnh đề 1.2. Nếu I là khoảng độc lập của R và f(x) là hàm lồi trên I,
thì f Lipschitz địa phương.
Chứng minh. Lấy K là tập compact trong I. Ta có thể chọn a < b < x <
y < c < d. Áp dụng tính chất (1.4), ta có
f(b) −f(a)
b −a

f(y) − f(x)
y −x

f(d) −f(c)
d −c
.
7
Để kết thúc chứng minh Mệnh đề, ta chọn
M
K
= max




f(b) −f(a)
b −a



,




f(d) −f(c)
d −c




.
Mệnh đề 1.3. Cho I là một khoảng và f : I −→ R là hàm khả vi. Khi đó
i)f là hàm lồi nếu và chỉ nếu
f(x) ≥ f(a) + f

(a)(x −a); x, a ∈ I.
ii) f là hàm lồi chặt nếu và chỉ nếu
f(x) > f(a) + f

(a)(x −a); x, a ∈ I, x = a.
Chứng minh. i) Trước tiên ta giả sử f là hàm lồi. Lấy x, a ∈ I, x = a và
λ ∈ (0, 1]. Vì f là hàm lồi nên ta có
f(a + λ)(x −a) = f((1 − λ)a + λx)
≤ (1 − λ)f(a) + λf(x) = f(a) + λ(f(x) −f(a)).
Vì vậy
f(a + λ)(x −a) − f(a)
λ(x −a)
(x −a) ≤ f(x) − f(a).
Khi λ → 0 ta có f

(a)(x−a) ≤ f(x)−f(a). Tiếp theo, ta cố định x, y ∈ I
và λ ∈ [0, 1]. Theo giả thiết

f(x) ≥ f((1 − λ)x + λy) + f

(1 −λx + λy)λ(x − y).

f(y) ≥ f((1 − λ)x + λy) −f

((1 −λ)x + λy)(1 − λ)(x −y).
Nhân hai vế bất đẳng thức đầu với 1 − λ và bất đẳng thức thứ hai với λ
sau đó cộng từng vế của hai bất đẳng thức. Ta được
(1 −λ)f(x) + λf(y) ≥ f((1 − λ)x + λy),
8
Từ đó suy ra f là hàm lồi.
ii) Giả sử f là hàm lồi chặt. Chứng minh tương tự (i) ta được
f(x) −f(a) >
f(a + λ(x −a))
λ
).
Theo câu (i) ta có,
f(a + λ(x −a)) − f(a)λ ≥ f

(a)(x −a).
Do đó suy ra f(x) > f(a) + f

(a)(x − a). Phần đủ của (ii) chứng minh
tương tự như đối với (i).
Mệnh đề 1.4. Cho I là một khoảng và f : I −→ R là hàm khả vi bậc hai
sao cho f

(x) ≥ 0 (hoặc f


(x) > 0),∀x ∈ I. Chứng minh rằng f là hàm
lồi hoặc lồi chặt.
Chứng minh. Áp dụng công thức Taylor, với ∀x, a ∈ I, tồn tại ξ giữa x và
a sao cho
f(x) = f(a) +
x −a
1!
f(a) +
(x −a)
2
2!
f

(ξ) ≥ f(a) + (x −a)f

(a)
Áp dụng Mệnh đề 1.3, suy ra điều phải chứng minh.
Mệnh đề 1.5. Cho f : I −→ R là hàm lồi. Chứng minh rằng f có đạo
hàm một phía hữu hạn, hơn nữa nếu x và y, với x ≤ y là điểm trong của
I, khi đó
f

(x

) ≤ f

(x
+
) ≤ f


(y

) ≤ f

(y
+
)
Chứng minh. Cho a là điểm trong của I, chọn x
1
, x
2
, y ∈ I sao cho x
1
<
x
2
< a < y. Khi đó, theo (1.4), ta có
f(x
1
) −f(a)
x
1
− a

f(x
2
) −f(a)
x
2
− a


f(y) − f(a)
y −a
(1.7)
Do đó f

(a

) = lim
x→a

f(x) −f(a)
x −a
tồn tại, hơn nữa giới hạn lớn nhất

f(y) − f(a)
y −a
.
Do đó f

(a

) hữu hạn.
Tương tự ta có f

(a
+
) tồn tại, hữu hạn và f

(a


) ≤ f

(a
+
) (theo (1.7))
9
Mệnh đề 1.6. Giả sử f : I −→ R là hàm lồi. Khi đó tập hợp các điểm
mà tại đó hàm f không khả vi nhiều nhất là đếm được.
Chứng minh. Đặt ϕ là tập các điểm x ∈ I sao cho f không khả vi tại
x. Ta liên kết các điểm x ∈ ϕ là điểm trong khoảng mở khác rỗng I
x
=
(f

(x

), f

(x
+
)). Do đó ta có I
x
∩ I
y
= ∅, x = y.
Tiếp theo, với x ∈ ϕ, ta có r
x
∈ I
x

∩ Q. Có một và chỉ một ánh xạ
ϕ  x −→ r
x
∈ Q.Điều đó chứng tỏ tập ϕ đếm được.
Ví dụ trên cung cấp một cách kiểm tra quan trọng về việc xác định các
điểm cực trị của hàm số thực trong C
2
.
Mệnh đề 1.7. Cho f là hàm lồi khả vi bậc 2 trên khoảng mở I sao cho f

liên tục. Giả thiết rằng ∃a ∈ I sao cho f

(a) = 0, f

(a) > 0. Chứng minh
rằng a là điểm cực tiểu chặt của f.
Chứng minh. Theo công thức Taylor. Ta có
f(x) = f(a) + f

(a)(x −a) +
f

(a)
2
(x −a)
2
+ o((x −a)
2
)
= f(a) +

f

(a)
2
(x −a)
2
+ o((x −a)
2
)
Khi x → a thì ∃δ > 0 sao cho với ∀x ∈ I ∩(a −δ, a + δ),
f(x) −f(a) ≥
f

(a)
4
(x −a)
2
Do đó a là điểm cực tiểu chặt của f.
10
Chương 2
Một số bất đẳng thức và cực trị của
các hàm lồi
Chương này trình bày những ứng dụng của bất đẳng thức Jensen và
bất đẳng thứ Karamata giải một số bài toán về bất đẳng thức và cực trị.
Nội dung của chương này được hình thành chủ yếu trên cơ sở các tài liệu
[1]-[9].
2.1 Bất đẳng thức Jensen
Định lý 2.1. Giả sử f là hàm lồi trên khoảng J. Nếu x
1
, x

2
, , x
n
∈ J và
λ
1
, λ
2
, , λ
n
∈ [0, 1], λ
1
+ λ
2
+ + λ
n
= 1, thì
f

n

k=1
λ
k
x
k


n


k=1
λ
k
f(x
k
). (2.1)
Nếu f là hàm lõm thì bất đẳng thức (2.1) có chiều ngược lại.
Chứng minh. Chúng ta sẽ chứng minh bất đẳng thức (2.1) bằng phương
pháp quy nạp. Với n = 2 bất đẳng thức đúng. Giả sử bất đẳng thức đã
cho đúng với n = m, ta chứng minh nó đúng với n = m + 1. Ta có
f

m+1

k=1
λ
k
x
k

= f

m

k=1
λ
k
x
k
+ λ

m+1
x
m+1

= f

λ
m+1
x
m+1
+ (1 −λ
m+1
)
1
1 −λ
m+1
m

k=1
λ
k
x
k

11
≤ λ
m+1
f(x
m+1
) + (1 −λ

m+1
)f

1
1 −λ
m+1
m

k=1
λ
k
x
k

= λ
m+1
f(x
m+1
) + (1 −λ
m+1
)f

m

k=1
λ
k
1 −λ
m+1
x

k

≤ λ
m+1
f(x
m+1
) + (1 −λ
m+1
)
m

k=1
λ
k
1 −λ
m+1
f(x
k
)
= λ
m+1
f(x
m+1
) +
m

k=1
λ
k
f(x

k
) =
m+1

k=1
λ
k
f(x
k
).
Định lý được chứng minh.
Nhận xét 2.1. 1. Đối với hàm lồi chặt, dấu "=" trong bất đẳng thức
Jensen (2.1) xảy ra khi và chỉ khi x
1
= x
2
= = x
n
. Điều này có thể
được chứng minh bằng phương pháp quy nạp.
2. Nếu λ
1
= λ
2
= = λ
n
=
1
n
thì bất đẳng thức (2.1) có dạng

f

x
1
+ x
2
+ + x
n
n


f(x
1
) + f(x
2
) + + f(x
n
)
n
. (2.2)
3. Nếu f là hàm lõm thì công thức (2.1) và công thức (2.2) tương ứng có
dạng
f

n

k=1
λ
k
x

k


n

k=1
λ
k
f(x
k
). (2.3)
f

x
1
+ x
2
+ + x
n
n


f(x
1
) + f(x
2
) + + f(x
n
)
n

. (2.4)
2.2 Sử dụng hàm lồi chứng minh các bất đẳng thức
cơ bản đối với dãy số
Bất đẳng thức Jensen có nhiều ứng dụng khác nhau. Mục này của luận
văn trình bày ứng dụng quan trọng là chứng minh các bất đẳng thức cơ
bản.
12
2.2.1 Bất đẳng thức AM-GM với trọng và bất đẳng thức AM-
GM
Định lý 2.2. Giả sử x
1
, x
2
, , x
n
≥ 0, λ
1
, λ
2
, , λ
n
> 0, sao cho λ
1

2
+
+ λ
n
= 1. Khi đó
λ

1
x
1
+ λ
2
x
2
+ + λ
n
x
n
≥ x
λ
1
1
x
λ
2
2
x
λ
n
n
. (2.5)
Đẳng thức xảy ra khi và chỉ khi x
1
= x
2
= = x
n

.
Chứng minh. Chúng ta có thể giả thiết rằng x
1
, x
2
, , x
n
> 0. Xét hàm số
f(x) = ln x, x ∈ (0, +∞). Vì f là hàm lõm ngặt trên (0, ∞), nên
ln(λ
1
x
1
+ λ
2
x
2
+ + λ
n
x
n
) ≥ λ
1
ln x
1
+ λ
2
ln x
2
+ + λ

n
ln x
n
,
hay
ln(λ
1
x
1
+ λ
2
x
2
+ + λ
n
x
n
) ≥ ln(x
λ
1
1
x
λ
2
2
x
λ
n
n
). (2.6)

Vì f(x) = ln x là hàm đồng biến trên (0, +∞), nên từ (2.6) suy ra
λ
1
x
1
+ λ
2
x
2
+ + λ
n
x
n
≥ x
λ
1
1
x
λ
2
2
x
λ
n
n
.
Định lý được chứng minh.
Như một hệ quả, nếu trong (2.6) cho λ
1
= λ

2
= λ
n
, ta có bất đẳng
thức AM-GM sau đây.
Định lý 2.3. ( Bất đẳng thức AM-GM). Nếu x
1
, x
2
, , x
n
≥ 0, thì
x
1
+ x
2
+ + x
n
n

n

x
1
x
2
x
n
. (2.7)
2.2.2 Bất đẳng thức trung bình lũy thừa

Định nghĩa 2.1. Giả sử x
1
, x
2
, , x
n
, λ
1
, λ
2
, , λ
n
> và sao cho λ
1

2
+
+ λ
n
= 1. Với mỗi t ∈ R, t = 0, trung bình lũy thừa M
t
cấp t được xác
định theo công thức
M
t
=

λ
1
x

λ
1
+ λ
2
x
λ
2
+ + λ
n
x
λ
n
n

1
t
.
13
Một số trường hợp riêng hay gặp:
M
1
=
λ
1
x
λ
1
+ λ
2
x

λ
2
+ + λ
n
x
λ
n
n
(2.8)
là trung bình cộng có trọng (WAM: weighted arithmetic mean)
M
−1
=
n
λ
1
x
1
+
λ
2
x
2
+ +
λ
n
x
n
(2.9)
được gọi là trung bình điều hòa có trọng (WHM: weighted harmonic mean)

M
2
=

λ
1
x
λ
1
+ λ
2
x
λ
2
+ + λ
n
x
λ
n
n
(2.10)
là trung bình căn bậc hai có trọng (WRMS: weighted root mean square).
Sử dụng quy tắc Hopital có thể nhận được các công thức sau
M
0
= lim
t→0
M
t
= x

λ
1
1
x
λ
2
2
x
λ
n
n
, (2.11)
M

= lim
t→∞
M
t
= max{x
1
,
2
, , x
n
}, (2.12)
M
−∞
= lim
t→−∞
M

t
= min{x
1
,
2
, , x
n
} (2.13)
Định lý 2.4. Giả sử x
1
, x
2
, , x
n
, λ
1
, λ
2
, , λ
n
> và sao cho λ
1
+ λ
2
+
+ λ
n
= 1. Nếu t và s là các số thực khác không và sao cho s < t, thì
M
s

≤ M
t
. (2.14)
Chứng minh. Nếu 0 < s < t, hoặc s < 0 < t, thì bất đẳng thức (2.14)
được suy ra từ bất đẳng thức Jensen với hàm lồi chặt f(x) = x
t/s
. Thật
vậy, nếu a
1
, a
2
, , a
n
, λ −1, λ
2
, , λ
n
và λ
1
+ λ
2
+ + λ
n
= 1, thì

λ
1
a
1
+ λ

2
a
2
+ + λ
n
a
n
n

t/s

λ
1
a
t/s
1
+ λ
2
a
t/s
2
+ + λ
n
a
t/s
n
n
.
Bằng cách chọn a
1

= x
s
1
, a
2
= x
s
2
, , a
n
= x
s
2
, ta có bất đẳng thức (2.14).
Nếu s < t < 0, thì 0 < −t < −s và sử dụng bất đẳng thức (2.14) với
1
x
1
,
1
x
2
, ,
1
x
n
, ta thấy

λ
1

(
1
x
1
)
−t
+ λ
2
(
1
x
2
)
−t
+ + λ
n
(
1
x
n
)
−t
n

1
−t
14


λ

1
(
1
x
1
)
−s
+ λ
2
(
1
x
2
)
−s
+ + λ
n
(
1
x
n
)
−s
n

1
−s
.
Bất đẳng thức cuối cùng này có thể được viết dưới dạng (2.14).
Nhận xét 2.2. Nếu t < 0 < s, thì M

t
≤ M
0
≤ M
s
. Chúng ta cũng có các
bất đẳng thức cổ điển sau đây:
M
−∞
≤ M
−1
≤ M
0
≤ M
1
≤ M
2
≤ M

,
tức là
min{x
1
, x
2
, , x
n
} ≤
n
λ

1
x
1
+
λ
2
x
2
+ +
λ
n
x
n
≤ x
λ
1
1
x
λ
2
2
x
λ
n
n

λ
1
x
λ

1
+ λ
2
x
λ
2
+ + λ
n
x
λ
n
n
≤ max{x
1
, x
2
, , x
n
}.
2.2.3 Bất đẳng thức H
˝
older
Định lý 2.5. Nếu p, q là các số thực lớn hơn 1 và sao cho
1
p
+
1
q
= 1 và
a

1
, a
2
, , a
n
, b
1
, b
2
, , b
n
là các số thực hoặc phức thì
n

i=1
|a
i
||b
i
| ≤

n

i=1
|a
i
|
p

1

p

n

i=1
|b
i
|
q

1
q
. (2.15)
Chứng minh. Chúng ta có thể giả thiết |a
k
| > 0, k = 1, 2, , n. Hàm số
f(x) = x
q
là hàm lồi chặt trên (0, ∞), nên áp dụng bất đẳng thức Jensen
ta có

n

k=1
λ
k
x
k

q


n

k=1
λ
k
x
q
k
, (2.16)
trong đó x
1
, x
2
, , x
n
, λ
1
, λ
2
, , λ
n
), λ
1
+ λ
2
+ + λ
n
= 1. Đặt
A =

n

k=1
|a
k
|
p
, λ
k
=
1
A
|a
k
|
p
, x
k
=
1
λ
k
|a
k
|||b
k
|.
Thay các đại lượng trên đây vào vế phải và vế trái của (2.16) ta được bất
đẳng thức (2.15).
15

Nhận xét 2.3. 1. Từ điều kiện xảy ra dấu "=" trong bất đẳng thức Jensen
suy ra dấu "=" trong bất đẳng thức (2.15) xảy ra khi và chỉ khi
|a
1
|
p
|b
1
|
q
=
|a
2
|
p
|b
2
|
q
=
|a
n
|
p
|b
n
|
q
.
2. Nếu p = q = 2, thì bất đẳng thức H

˝
older chính là bất đẳng thức
Cauchy-Schwarz

n

k=1
|a
k
||b
k
|

2


n

k=1
|a
k
|
2

n

k=1
|b
k
|

2

.
Dấu đẳng thức xảy ra khi và chỉ khi
|a
1
|
|b
1
|
=
|a
2
|
|b
2
|
=
|a
n
|
|b
n
|
.
2.2.4 Bất đẳng thức tam giác Minkowski
Định lý 2.6. Nếu p ≥ 1 và a
1
, a
2

, , a
n
, b
1
, b
2
, , b
n
≥ 0, thì

n

k=1
(a
k
+ b
k
)
p

1
p


n

k=1
a
p
k


1
p

n

k=1
b
p
k

1
p
. (2.17)
Chứng minh. Chúng ta có thể giả thiết a
k
≥ 0, k = 1, 2, , n. Hàm f(x) =

1 + x
1
p

p
, x ∈ (0, ∞) là hàm lõm ngặt vì
f

(x) =
1 −p
p


1 + x
1
p

p−2
.x
1
p
−2
< 0.
Áp dụng bất đẳng thức Jensen ta có

1 +

n

k=1
λ
k
x
k

1
p

p

n

k=1

λ
k

1 + x
1
k
k

p
. (2.18)
với x
1
, x
2
, , x
n
, λ
1
, λ
2
, , λ
n
> 0 và λ
1
+ λ
2
+ + λ
n
= 1, trong đó
x

1
, x
2
, , x
n
, λ
1
, λ
2
, , λ
n
> 0 và λ
1
+ λ
2
+ + λ
n
= 1. Ký hiệu
A =
n

k=1
a
p
k
, λ
k
=
a
p

k
A
, k = 1, 2, , n.
Đưa các biểu thức trên đây vào hai vế của bất đẳng thức (2.18) ta được
bất đẳng thức (2.17).
16
Nhận xét 2.4. 1. Dấu đẳng thức trong (2.17) xảy ra khi và chỉ khi
b
1
a
1
=
b
2
a
2
= =
b
n
a
n
.
2. Khi p = 2 ta có bất đẳng thức có tên gọi bất đẳng thức tam giác sau đây




n

k=1

(a
k
+ b
k
)
2





n

k=1
a
2
+




n

k=1
b
2
.
2.2.5 Bất đẳng thức Young
Trong (2.5) ta chọn n = 2, x
1

= A, x
2
= B, λ
1
=
1
p
, λ
2
=
1
q
, với
p, q > 0 và
1
p
+
1
q
= 1. Ta có
A
1
p
B
1
q

A
p
+

B
q
, (2.19)
Trong (2.19) chọn
A = a
p
, B = b
p
ta có bất đẳng thức Young ( William Young, 1863-1942):
ab ≤
a
p
p
+
b
q
q
. (2.20)
2.3 Một số bài toán ứng dụng bất đẳng thức Jensen
2.3.1 Đại số và lượng giác
Bài toán 2.1. Cho x là một số thực thỏa mãn 0 < x <
π
4
. Chứng minh
rằng
(sinx)
sinx
< (cosx)
cosx
.

Lời giải. Ta tìm một cận dưới dương của ln(cosx) − tanxln(sinx) với
0 < x <
π
4
. Với tất cả số thực a,b và λ với a > 0, b > 0, λ ∈ (0, 1),vì hàm
logarit là hàm lõm nên ta có
ln(λa + (1 −λ)b) > λlna + (1 −λ)lnb.
17
Cho x ∈ R với 0 < x <
π
4
và định nghĩa a = sinx, b = sinx + cosx, λ =
tanx. Bất đẳng thức trên cho ta
ln(cosx) > tanxln(sinx) + (1 − tanx)ln(sinx + cosx).
Ta thấy (1 − tanx)ln(sinx + cosx) > 0, vì tanx < 1, sinx + cosx =

2cos(
π
4
− x) > 1.
Bài toán 2.2. So sánh tan(sinx) và sin(tanx) với x ∈ (0,
π
2
).
Lời giải. Đặt f(x) = tan(sinx) − sin(tanx). Ta có
f

(x) =
cosx
cos

2
(sinx)

cos(tanx)
cos
2
x
=
cos
3
x −cos(tanx).cos
2
(sinx)
cos
2
x.cos
2
(tanx)
.
Chọn 0 < x < arctan
π
2
. Vì hàm cosin là hàm lõm trên (0,
π
2
) nên ta có
3

cos(tanx).cos
2

(sinx) <
1
3
[cos(tanx) + 2cos(sinx)]
≤ cos

tanx + 2sinx
3

< cosx.
Bất đẳng thức cuối được suy ra từ

tanx + 2sinx
3


=
1
3

1
cos
2
x
+ 2cosx


3

1

cos
2
x
.cosx.cosx = 1.
Điều này chứng tỏ rằng cos
3
x −cos(tanx).cos
2
(sinx) > 0 hay f

(x) > 0.
Do đó f là hàm số tăng trên [0, arctan
π
2
]. Kết thúc chứng minh, ta thấy
rằng
tan

sin(arctan
π
2
)

= tan
π/2

1 + π
2
/4
> tan

π
4
= 1.
Ta dễ dàng thấy rằng, nếu x ∈ [arctan
π
2
,
π
2
] thì tan(sinx) > 1 và do đó
f(x) > 0.
Bài toán 2.3. Cho f : [0, 1] −→ R là hàm liên tục sao cho f(0) = 0.
Chứng minh rằng tồn tại một hàm liên tục lõm g : [0, 1] −→ R sao cho
g(0)=0 và g(x) ≥ f(x), ∀x ∈ [0, 1].
18
Lời giải. Cho M = max
x∈[0,1]
|f(x)|. Định nghĩa dãy (x
n
) sao cho x
0
=
1, 0 < x
n
< 3
−1
x
n−1
thỏa mãn |f(x)| <
M

2
n
, với 0 < x < x
n
. Mặt khác với
bất kì x ∈ (0, 1], tồn tại duy nhất n ≥ 0 sao cho x = λx+(1−λx)x
n
, λ
x

[0, 1]. Định nghĩa hàm liên tục g : [0, 1] −→ [0, 1] sao cho g(0) = 0 và
g(x) = λ
x
M
2
n
+ (1 −λ
x
)
M
2
n−1
, ∀x ∈ (0, 1].
Ta có g ≥ f và
g(x
n
) −g(x
n+1
)
x

n
− x
n+1
=
M/2
n
x
n
− x
n+1
>
M/2
n−1
x
n
− x
n+1
=
g(x
n−1
) −g(x
n
)
x
n−1
− x
n
,
do đó g là hàm lõm.
Bài toán 2.4. Cho f : [0, 1] −→ R là hàm lõm tăng thỏa mãn f(0) = 0

và f(1) = 1. Chỉ ra rằng f(x)f
−1
(x) ≤ x
2
với 0 ≤ ∀x ≤ 1.
Lời giải. Cho 0 < a ≤ b ≤ 1. Đặt t =
b −a
a
và sử dụng định nghĩa hàm
lõm, ta được
f(a) ≥
af(b) + (b −a)f(0)
b
=
af(b)
b
.
Nếu a = x và b = 1 thì bất đẳng thức trên trở thành f(x) ≥ x. Mà x =
f
−1
(f(x)) ≥ f
−1
(x)(do f
−1
là hàm tăng). Với b = x = 0 và a = f
−1
(x)
ta được
x = f(f
−1

(x)) ≥
f
−1
(x)f(x)
x
.
Do đó f(x)f
−1
(x) ≤ x
2
, ∀x ∈ [0, 1].
Bài toán 2.5. Cho a, b, u, v là các số không âm sao cho a
5
+ b
5
≤ 1 và
u
5
+ v
5
≤ 1. Chứng minh rằng
a
2
u
3
+ b
2
v
3
≤ 1.

Lời giải. Theo bất đẳng thức trung bình của số học-hình học,ta có
2a
5
+ 3u
5
5
=
a
5
+ a
5
+ u
5
+ u
5
+ u
5
5
≥ (a
10
u
15
)
1
5
= a
2
u
3
.

19
Tương tự đối với 2 số b, v ta có
2b
5
+ 3v
5
5
=
b
5
+ b
5
+ v
5
+ v
5
+ v
5
5
≥ (b
10
v
15
)
1
5
= b
2
v
3

.
Cộng từng vế ta được điều phải chứng minh.
Bài toán 2.6. Cho x, y ∈ (0,

π/2) với x = y. Chứng minh rằng
ln
2
1 + sin(xy)
1 −sin(xy)
< ln
1 + sin(x
2
)
1 −sin(x
2
)
.ln
1 + sin(y
2
)
1 −sin(y
2
)
.
Lời giải. Cho t ∈ (−∞, ln
π
2
).
Định nghĩa hàm số f(t) = ln(ln1 +
sine

t
1 −sine
t
). Ta thấy f là hàm lồi chặt.
Thật vậy
f

(t) =
2e
t
e
2f(t)
cos
2
e
t

(cose
t
+ e
t
sine
t
)ln

1 + sine
t
1 −sine
t


− 2e
t

.
Đặt
g(u) = (cosu + usinu)ln

1 + sinu
1 −sinu

− 2u.
Ta có g(0) = 0 và
g

(u) = ucosuln

1 + sinu
1 −sinu

+ 2utanu > 0,
Với 0 < u < π/2 suy ra g(u) > 0. Điều này dẫn đến f

(t) > 0, hay f(t)
là hàm lồi chặt trên (−∞, ln(π/2)). Do đó
ln
2
1 + sin(xy)
1 −sin(xy)
= e
2f(lnx+lny)

< e
f(2lnx)+f(2lny)
= ln
1 + sin
2
1 −sinx
2
.ln
1 + siny
2
1 −siny
2
,
với x, y ∈ (0,

π/2), x = y.
Theo khẳng định của định lí Hilbert, giả sử p > 1, p

=
p
p −1
. Ta xét
A :=


n=1
a
p
n
, B :=



k=1
b
p

k
, với (a
n
)
n≥1
, (b
n
)
n≥1
là dãy các số không âm. Khi
đó


n=1


k=1
a
n
b
k
n + k
<
π

sin(π/p)
A
1/p
B
1/p

.
Dấu ” = ” xảy ra khi A = 0 hoặc B = 0.
20

×